Answer (B) is correct . If the labor costs for the plastic frames are reduced by $1, the composite unit contribution margin will be $27?{($10 – $2 – $2) + [($15 – $3 – $5) × 3]}. Hence, the new breakeven point is 144,444 units [4?units?× ($975,000 FC ÷ $27)].
Answer (A) is incorrect because The new breakeven point is 144,444 units. Answer (C) is incorrect because The new breakeven point is 144,444 units. Answer (D) is incorrect because The new breakeven point is 144,444 units.
|